Answer (D) is correct . When monetary outcomes and probabilities can be reasonably estimated, a payoff table can be constructed to determine the best course of action: Investment A Investment B Expected Expected Payoffs Probability Value Payoffs Probability Value $(20,000) 0.3 $ (6,000) $(20,000) 0.2 $ (4,000) (10,000) 0.1 (1,000) (10,000) 0.2 (2,000) ? 30,000 0.3 9,000 30,000 0.2 6,000 70,000 0.2 14,000 70,000 0.2 14,000 100,000 0.1 10,000 100,000 0.2 20,000 Total $26,000 Total $34,000
Answer (A) is incorrect because Investment B has a higher expected value than Investment A.
Answer (B) is incorrect because Investment C has a higher expected value than Investment A.
Answer (C) is incorrect because Investment B has a higher expected value than both Investment C and Investment?A.
|